Fix subscripting (r_12 -> r_{12}) in Serway and Jewett v8's 25.21.
[course.git] / latex / problems / Serway_and_Jewett_4_wking / problem28.38.T.tex
1 \begin{problem}
2 An electron that has an energy of approximately $6\U{eV}$ moves
3 between rigid walls $1.00\U{nm}$ apart.  Find \Part{a} the quantum
4 number $n$ for the energy state that the electron occupies
5 and \Part{b} the precise energy of the electron.
6 \end{problem} % based on 28.38
7
8 \begin{solution}
9 \Part{a}
10 The allowed energy levels for a particle in a box are (Equation 28.30)
11 \begin{align}
12   E_n &= \frac{h^2 n^2}{8mL^2} \;.
13 \end{align}
14 For an electron ($m=9.11\E{-31}\U{kg}$) in a box of length
15 $L=1\U{nm}$, this works out to
16 \begin{align}
17   E_1 &= 0.377\U{eV} \\
18   E_2 &= 1.51\U{eV} \\
19   E_3 &= 3.39\U{eV} \\
20   E_4 &= 6.02\U{eV} \\
21   E_5 &= 9.41\U{eV}
22 \end{align}
23 So the electron is in the $\ans{n=4}$ state.
24
25 \Part{b}
26 The precise energy is $E_4 = \ans{6.02\U{eV}}$.
27 \end{solution}